Difference between revisions of "2010 AIME II Problems/Problem 1"

m (Solution)
m (Solution)
Line 4: Line 4:
 
== Solution ==
 
== Solution ==
 
If we include all the even digits for the greatest integer multiple, we find that it is impossible for it to be divisible by <math>3</math>, therefore by <math>36</math> as well.
 
If we include all the even digits for the greatest integer multiple, we find that it is impossible for it to be divisible by <math>3</math>, therefore by <math>36</math> as well.
The next logical try would be <math>8640</math>, which happens to be divisible by <math>36</math>. Thus <math>N = 8640 mod 1000 = \fbox{640}</math>
+
The next logical try would be <math>8640</math>, which happens to be divisible by <math>36</math>. Thus <math>N = 8640 \pmod {1000} = \boxed{640}</math>

Revision as of 10:58, 2 April 2010

Problem

Let $N$ be the greatest integer multiple of $36$ all of whose digits are even and no two of whose digits are the same. Find the remainder when $N$ is divided by $1000$.

Solution

If we include all the even digits for the greatest integer multiple, we find that it is impossible for it to be divisible by $3$, therefore by $36$ as well. The next logical try would be $8640$, which happens to be divisible by $36$. Thus $N = 8640 \pmod {1000} = \boxed{640}$